LSAT and Law School Admissions Forum

Get expert LSAT preparation and law school admissions advice from PowerScore Test Preparation.

User avatar
 Albertlyu
  • Posts: 98
  • Joined: Jul 18, 2020
|
#82074
thanks, I am confused about the way answer choice D was phrased: "the Beverage was not the only cause of the ineffectiveness of the prescribed dosage", which implies that the Beverage is one of the causes, but from the second set of recommendations and results of their application(without knowing the 3rd recommendations or results), we can not know whether or not the beverage is the cause, it could just be the doc has prescribed the wrong drugs. please can anyone help?

thanks very much.

Albert
 Adam Tyson
PowerScore Staff
  • PowerScore Staff
  • Posts: 5153
  • Joined: Apr 14, 2011
|
#82086
That's not borne out by the results, Albert! The medication did prove to be effective after the doctor eliminated the beverage AND doubled the dose. So he was right that the dosage was too low, but the beverage was also causing a problem. Remember, doubling the dose didn't fix the problem, so the dosage alone wasn't the issue. Getting rid of the beverage but keeping the original dose didn't fix the problem either, so the beverage alone was not the problem. Only doing both did the trick, so that indicates that there were two problems that needed to be resolved.
User avatar
 Albertlyu
  • Posts: 98
  • Joined: Jul 18, 2020
|
#82430
Adam Tyson wrote:That's not borne out by the results, Albert! The medication did prove to be effective after the doctor eliminated the beverage AND doubled the dose. So he was right that the dosage was too low, but the beverage was also causing a problem. Remember, doubling the dose didn't fix the problem, so the dosage alone wasn't the issue. Getting rid of the beverage but keeping the original dose didn't fix the problem either, so the beverage alone was not the problem. Only doing both did the trick, so that indicates that there were two problems that needed to be resolved.

thanks, Adam for your reply! I think I know where I got wrong: I interpreted the question stem as how the 2nd set of recommendations and results supported the initial hypothesis WITHOUT KNOWING THE FINAL RESULTS, so stopped the beverage and maintained the initial dosage, and nothing happened.
User avatar
 blaisebayno
  • Posts: 25
  • Joined: May 24, 2022
|
#97614
Hi,

Just because the beverage OFTEN interferes with the medication, doesn't mean we should assume that it was doing so for the patient! In fact, the lower dosage's ineffectiveness both in the presence of the beverage and in its absence indicates that the beverage had NO effect on the medication's effectiveness!!! Someone PLEASE help me understand why I am incorrect for choosing C.
 Robert Carroll
PowerScore Staff
  • PowerScore Staff
  • Posts: 1787
  • Joined: Dec 06, 2013
|
#97935
blaisebayno,

It's true that just because the beverage often inhibits the medication's effect doesn't mean it always does, but it actually appears to have done just that in this situation. When the dosage was doubled the first time, the symptoms remained, but when the dosage was doubled AND the patient stopped drinking the beverage, as happened in the last situation, the symptoms disappeared. That indicates pretty strongly that the beverage was inhibiting the effect - double the dosage worked when the patient stopped drinking the beverage and didn't work when they kept drinking the beverage. When I remove a supposed cause and its supposed effect goes away, that's a good indication it really was causing that effect!

Further, answer choice (C) is saying the beverage was responsible, but in your post you think the beverage wasn't responsible. I'm unclear why you picked that answer if it's saying the opposite of what you believed. The second set of recommendations (stop the beverage, but return to the initial dose) indicates the dosage was a contributing factor to ineffectiveness, and certainly doesn't argue in favor of the beverage. As I said, the first and last situations indicate the beverage is a likely partial cause of the ineffectiveness.

Robert Carroll

Get the most out of your LSAT Prep Plus subscription.

Analyze and track your performance with our Testing and Analytics Package.